Completed rec9 solutions
[course.git] / latex / problems / Young_and_Freedman_12 / problem28.60.tex
1 \begin{problem*}{28.60}
2 Figure~28.54 shows an end view of two long, parallel wires
3 perpendicular to the $xy$-plane, each carrying a current $I$ but in
4 opposite directions.  \Part{a} Copy the diagram, and draw vectors to
5 show the \vect{B} field of each wire and the net \vect{B} field at a
6 point $P$.  \Part{b} Derive the expression for the magnitude of
7 \vect{B} at any point on the $x$-axis in terms of the $x$-coordinate
8 of the point.  What is the direction of \vect{B}?  \Part{c} Graph the
9 magnitude of \vect{B} at points on the $x$-axis.  \Part{d} At what
10 value of $x$ is the magnitude of \vect{B} a maximum?  \Part{e} What is
11 the magnitude of \vect{B} when $x\gg a$?
12 \end{problem*}
13
14 \begin{nosolution}
15 \begin{center}
16 \begin{asy}
17 import graph;
18 import Mechanics;
19
20 xaxis("$x$");
21 yaxis("$y$");
22
23 real u = 1cm;
24 real a = 1u;
25 real x = 3u;
26 pen ipen = red+blue;
27
28 Vector Vt = Vector(Scale((0,a)), phi=90, ipen);
29 Vector Vb = Vector(Scale((0,-a)), phi=-90, ipen);
30 Vt.draw();
31 Vb.draw();
32
33 dot("P", Scale((x,0)), N);
34
35 label("$a$", align=W, Scale((0,a/2)));
36 label("$a$", align=W, Scale((0,-a/2)));
37 label("$x$", align=S, Scale((x/2,0)));
38 \end{asy}
39 \end{center}
40 \end{nosolution}
41
42 \begin{solution}
43 \Part{a}
44 \begin{center}
45 \begin{asy}
46 import graph;
47 import Mechanics;
48 import ElectroMag;
49
50 xaxis("$x$");
51 yaxis("$y$");
52
53 real u = 1cm;
54 real a = 1u;
55 real x = 3u;
56 pen ipen = red+blue;
57
58 Vector Vs[];
59 Vs.push(Vector(Scale((0,a)), phi=90, ipen));
60 Vs.push(Vector(Scale((0,-a)), phi=-90, ipen));
61 Vs.push(BField((x,0), dir=degrees((x,-a))+90)); // From top wire
62 Vs.push(BField((x,0), dir=degrees((x,a))-90)); // From bottom wire
63 Vs.push(BField((x,0)));  // Net
64
65 for (int i=0; i<Vs.length; i+=1) {
66   Vs[i].draw();
67 }
68
69 dot("P", Scale((x,0)), N);
70
71 label("$a$", align=W, Scale((0,a/2)));
72 label("$a$", align=W, Scale((0,-a/2)));
73 label("$x$", align=S, Scale((x/2,0)));
74 \end{asy}
75 \end{center}
76
77 \Part{b}
78 The magnitude of magnetic field from each wire at $P$ is
79 \begin{equation}
80   B_w = \frac{\mu_0 I}{2\pi r}
81     = \frac{\mu_0 I}{2\pi\sqrt{x^2+a^2}} \;.
82 \end{equation}
83 The net magnetic field is the sum of the horizontal components, since
84 the vertical components cancel.
85 \begin{equation}
86   B = 2 B_w \cos\theta
87     = 2\cdot\frac{\mu_0 I}{2\pi\sqrt{x^2+a^2}}\cdot\frac{a}{\sqrt{x^2+a^2}}
88     = \ans{\frac{\mu_0 I a}{\pi(x^2+a^2)}} \;,
89 \end{equation}
90 where $\cos\theta=a/r$ comes from
91 \begin{center}
92 \begin{asy}
93 import Mechanics;
94 import ElectroMag;
95
96 real u = 1cm;
97 real a = 1u;
98 real x = 3u;
99
100 Vector Bt = BField((x,0), dir=degrees((x,-a))+90); // From top wire
101 Angle theta1 = Angle((x,0)+dir(0), (x,0), (x,0)+dir(Bt.dir), "$\theta$");
102 Angle theta2 = Angle((0,0), (0,a), (x,0), "$\theta$");
103
104 theta1.draw();
105 theta2.draw();
106 draw((0,0)--(0,a)--(x,0)--cycle);
107 Bt.draw();
108 label("$r$", (x/2,a/2), NE);
109 label("$a$", (0,a/2), W);
110 \end{asy}
111 \end{center}
112
113 \Part{c}
114 \begin{center}
115 \begin{asy}
116 import graph;
117 size(5cm, 2cm, IgnoreAspect);
118
119 real a=1;
120 real B(real x){
121   return 1.0/(x**2 + 1);
122 }
123
124 xaxis("$x$");
125 yaxis("$B$");
126 draw(graph(B, -2*a, 2*a, n=333), red);
127 label("$a$", align=S, Scale((a,0)));
128 label("$-a$", align=S, Scale((-a,0)));
129 \end{asy}
130 \end{center}
131
132 \Part{d}
133 The magnitude of \vect{B} has a maximum at $\ans{x=0}$.
134
135 \Part{e}
136 For $x\gg a$, $x^2+a^2\approx x^2$, so
137 \begin{equation}
138   B = \frac{\mu_0 I a}{\pi(x^2+a^2)}
139     \approx \ans{\frac{\mu_0 I a}{\pi x^2}} \;.
140 \end{equation}
141 \end{solution}